Exercise Set 14
MONOPOLY


I. Objectives


II. Data


III. Questions

  1. Select values for the parameters: a = ________, F = _________. (Note: a should be less than 150.)
  2. Sketch the demand curve.
  3. Obtain the marginal revenue function. Sketch the MR curve.
  4. The slope of the MR curve is [ half / same as / twice ] the slope of the demand curve.
  5. Sketch the MC and AC curves.
  6. In order to maximize profits, the firm will produce ______ units of output and sell them at a price of $ _______ each. Explain how how you obtained these values. Indicate the optimal point on the graph.
  7. At the optimal point, we conclude that Price is [ greater than / equal to / less than ] Average Cost. Explain.
  8. The price charged by the firm is [ above / below / same as] the marginal revenue and [ above / below / same as ] the marginal cost.
  9. The firm's maximum profit is $ ________.
  10. If the firm's fixed costs fall, the firm [ will raise / will lower / will not change ] its price and it will produce [ more / less / the same ] output. Explain.
  11. If the firm's marginal costs rise, the firm [ will raise / will lower / will not change ] its price and it will produce [ more / less / the same ] output. Explain. [Hint: How do you interpret an increase in a?]

IV. Additional Questions

  1. A profit-maximizing monopolist produces the output at which

    (a) Price equals marginal cost
    (b) Marginal revenue equals marginal cost
    (c) Price equals average cost
    (d) Average cost is minimized

  2. A monopolist is currently producing 6 units of a good. The corresponding price, from the demand curve, is $30. At this output level, marginal revenue is $18/unit while marginal cost is $15/unit. In order to increase profits, the monopolist should

    (a) Reduce output and raise the price
    (b) Increase output and lower the price
    (c) Reduce output and lower the price
    (d) Increase output and price

  3. If a monopolist decides to maximize sales (revenue) instead of profits, it should produce an output level such that

    (a) MR = 0
    (b) MR = MC
    (c) P = MC
    (d) P = AC
    (e) None of the above is satisfied

  4. A monopolist currently produces 20 units of a good and sells them for $50 apiece. For the 20th unit, the marginal revenue is $24/unit while the marginal cost is $33/unit. In order to increase profits, the monopolist should

    (a) Reduce output and raise the price
    (b) Increase output and lower the price
    (c) Reduce output and lower the price
    (d) Increase output and price


Answers to questions in Section IV:

	1b	2b	3a	4a

EC102